7. Prove the following using induction. n Σk³ = 1³ + 2³ +3³ + ... + n³ = (1 + 2 + 3 + ... + n)² 13 23 k=1 .. [Hint: Recall that 1 + 2 + 3 + ··· + n = n(n+1) 2 ]

Algebra & Trigonometry with Analytic Geometry
13th Edition
ISBN:9781133382119
Author:Swokowski
Publisher:Swokowski
Chapter1: Fundamental Concepts Of Algebra
Section1.2: Exponents And Radicals
Problem 92E
icon
Related questions
Question

Discrete Mathematics

### Inductive Proof of Sum of Cubes Formula

**Problem:**
Prove the following using induction.

\[
\sum_{k=1}^{n} k^3 = 1^3 + 2^3 + 3^3 + \cdots + n^3 = \left(1 + 2 + 3 + \cdots + n\right)^2
\]

**Hint:**
Recall that \(1 + 2 + 3 + \cdots + n = \frac{n(n+1)}{2}\).

### Explanation of the Formula and Steps for Inductive Proof:

1. **Base Case:**
   For \(n = 1\):

   \[
   \sum_{k=1}^{1} k^3 = 1^3 = 1
   \]

   \[
   \left(1 + 2 + 3 + \cdots + 1\right)^2 = 1^2 = 1
   \]

   The base case holds true since both sides are equal to 1.

2. **Inductive Step:**
   Assume the formula holds for some integer \(n = k\), i.e.,

   \[
   \sum_{k=1}^{n} k^3 = \left( \frac{n(n+1)}{2} \right)^2
   \]

   Now show that the formula holds for \(n = k + 1\):

   Consider the sum \(\sum_{k=1}^{n+1} k^3\):

   \[
   \sum_{k=1}^{n+1} k^3 = \sum_{k=1}^{n} k^3 + (n+1)^3
   \]

   Using the inductive hypothesis:

   \[
   \sum_{k=1}^{n+1} k^3 = \left( \frac{n(n+1)}{2} \right)^2 + (n+1)^3
   \]

   Simplify and show that this equals \(\left( \frac{(n+1)(n+2)}{2} \right)^2\).

This proof technique ensures that the stated formula \(\sum_{k=1}^{n} k^3 = \left( \frac{n(n+1)}{2} \right)^2\) is
Transcribed Image Text:### Inductive Proof of Sum of Cubes Formula **Problem:** Prove the following using induction. \[ \sum_{k=1}^{n} k^3 = 1^3 + 2^3 + 3^3 + \cdots + n^3 = \left(1 + 2 + 3 + \cdots + n\right)^2 \] **Hint:** Recall that \(1 + 2 + 3 + \cdots + n = \frac{n(n+1)}{2}\). ### Explanation of the Formula and Steps for Inductive Proof: 1. **Base Case:** For \(n = 1\): \[ \sum_{k=1}^{1} k^3 = 1^3 = 1 \] \[ \left(1 + 2 + 3 + \cdots + 1\right)^2 = 1^2 = 1 \] The base case holds true since both sides are equal to 1. 2. **Inductive Step:** Assume the formula holds for some integer \(n = k\), i.e., \[ \sum_{k=1}^{n} k^3 = \left( \frac{n(n+1)}{2} \right)^2 \] Now show that the formula holds for \(n = k + 1\): Consider the sum \(\sum_{k=1}^{n+1} k^3\): \[ \sum_{k=1}^{n+1} k^3 = \sum_{k=1}^{n} k^3 + (n+1)^3 \] Using the inductive hypothesis: \[ \sum_{k=1}^{n+1} k^3 = \left( \frac{n(n+1)}{2} \right)^2 + (n+1)^3 \] Simplify and show that this equals \(\left( \frac{(n+1)(n+2)}{2} \right)^2\). This proof technique ensures that the stated formula \(\sum_{k=1}^{n} k^3 = \left( \frac{n(n+1)}{2} \right)^2\) is
Expert Solution
steps

Step by step

Solved in 3 steps with 2 images

Blurred answer
Recommended textbooks for you
Algebra & Trigonometry with Analytic Geometry
Algebra & Trigonometry with Analytic Geometry
Algebra
ISBN:
9781133382119
Author:
Swokowski
Publisher:
Cengage
College Algebra
College Algebra
Algebra
ISBN:
9781938168383
Author:
Jay Abramson
Publisher:
OpenStax